Quantcast
  • Register
PhysicsOverflow is a next-generation academic platform for physicists and astronomers, including a community peer review system and a postgraduate-level discussion forum analogous to MathOverflow.

Welcome to PhysicsOverflow! PhysicsOverflow is an open platform for community peer review and graduate-level Physics discussion.

Please help promote PhysicsOverflow ads elsewhere if you like it.

News

PO is now at the Physics Department of Bielefeld University!

New printer friendly PO pages!

Migration to Bielefeld University was successful!

Please vote for this year's PhysicsOverflow ads!

Please do help out in categorising submissions. Submit a paper to PhysicsOverflow!

... see more

Tools for paper authors

Submit paper
Claim Paper Authorship

Tools for SE users

Search User
Reclaim SE Account
Request Account Merger
Nativise imported posts
Claim post (deleted users)
Import SE post

Users whose questions have been imported from Physics Stack Exchange, Theoretical Physics Stack Exchange, or any other Stack Exchange site are kindly requested to reclaim their account and not to register as a new user.

Public \(\beta\) tools

Report a bug with a feature
Request a new functionality
404 page design
Send feedback

Attributions

(propose a free ad)

Site Statistics

205 submissions , 163 unreviewed
5,047 questions , 2,200 unanswered
5,345 answers , 22,709 comments
1,470 users with positive rep
816 active unimported users
More ...

  Naive question about the S-matrix

+ 2 like - 0 dislike
1928 views

In quantum field theory, the elements of the S-matrix are defined as the amplitude describing the transition from an initial $n$-particle state (the "in" state) to an final $m$-particle state: \begin{equation} S_{fi} = \langle \mathbf{q}_1,\dots,\mathbf{q}_m; \text{out} | \mathbf{p}_1,\dots,\mathbf{p}_n ; \text{in} \rangle \tag{1} \end{equation} To me it seems that this equation only makes sense if the amount of "in" particles is equal to the amount of "out" particles (i.e. $m=n$) otherwise we can not take the inner product. For instance, if $m=2$ and $n=3$, then we can write equation $(1)$ as: \begin{equation} \begin{pmatrix} \mathbf{q}_1 & \mathbf{q}_2 \end{pmatrix} \begin{pmatrix} \mathbf{p}_1 \\ \mathbf{p}_2 \\ \mathbf{p}_3 \end{pmatrix} =\ ? \end{equation} which is undefined. Therefore, my question is how to interpret equation $(1)$ if $m \neq n$?

I must admit that I have never studied the S-matrix in quantum mechanics. Therefore, I apologize in advance if this is a naive question.

This post imported from StackExchange Physics at 2014-04-24 02:32 (UCT), posted by SE-user Hunter
asked Jan 18, 2014 in Theoretical Physics by Hunter (520 points) [ no revision ]

3 Answers

+ 4 like - 0 dislike

The Hilbert space of the theory $\cal H$ can be viewed simultaneously as a Fock space in two different manners: $${\cal H} = {\cal F}_{symm}(K_{in}) = {\cal F}_{symm}(K_{out})\:.$$ Where $K_{in}$ and $K_{out}$ are the one-particle space of ingoing and outgoing free particles and I am assuming that the particles are Bosons for the sake of simplicity. Above: $${\cal F}_{symm}(K) = {\mathbb C}\oplus K \oplus (K\otimes K)_{symm} \oplus (K\otimes K\otimes K)_{symm}\oplus \cdots $$

The point is now that, in general $K_{in}$ include not only vectors of $K_{out}$, but even of $(K_{out}\otimes K_{out})_{symm}$, $(K_{out}\otimes K_{out}\otimes K_{out})_{symm}$, and so on. In other words it is false that, for instance $K_{in} \not\perp (K_{out}\otimes \cdots (k \:times)\cdots \otimes K_{out})_{symm}$ for $k>1$.

This is the mathematical translation of the fact that an ingoing free particle, in the asymptotic past (when the interactions are switched off), due to interactions at finite time, may give rise to many free particles in the asymptotic future (when the interactions are again switched off). In general the number of particles is not preserved due to the interactions passing from $t=-\infty$ to $t=+\infty$.

In general, for every value of $n$ and $m$:

$(K_{in}\otimes \cdots (n \:times)\cdots \otimes K_{in})_{symm} \not\perp (K_{out}\otimes \cdots (m \:times)\cdots \otimes K_{out})_{symm}$.

These relations can be written using vectors:

$$\langle\psi^{(out)}_{1}\cdots \psi^{(out)}_{m}| \psi^{(in)}_{1}\cdots \psi^{(in)}_{n}\rangle \neq 0 \quad \mbox{for generic $n\neq m$,}$$

where, for instance

$$ |\psi^{(out)}_{1}\cdots \psi^{(out)}_{m}\rangle \in (K_{out}\otimes \cdots (m \:times)\cdots \otimes K_{out})_{symm}$$

is the generic symmetrised state made of $m$ outgoing particles with single states $\psi^{(out)}_{1},\cdots, \psi^{(out)}_{m}\in K_{out}$, not necessarily pairwise different.

This post imported from StackExchange Physics at 2014-04-24 02:32 (UCT), posted by SE-user V. Moretti
answered Jan 18, 2014 by Valter Moretti (2,085 points) [ no revision ]
Thanks for your answer and +1. I should really one day buy "PCT, Spin and Statistics, and All That" (as you have recommended me in another post), because one day I hope to understand this as well as you do.

This post imported from StackExchange Physics at 2014-04-24 02:32 (UCT), posted by SE-user Hunter
I became a mathematician but I had beforehand learned these things using "hands and feet" as a physicist. I think that is the correct way to understand QFT. Later, one can pass to more formal books "PCT, Spin and Statistics, and All That" or Haag's textbook...so, do not worry you are doing the right thing, for the moment at least.

This post imported from StackExchange Physics at 2014-04-24 02:32 (UCT), posted by SE-user V. Moretti
@V.Moretti I couldn't agree more. Also, to which textbook of Haag's are you referring?

This post imported from StackExchange Physics at 2014-04-24 02:32 (UCT), posted by SE-user joshphysics
Local Quantum Physics

This post imported from StackExchange Physics at 2014-04-24 02:32 (UCT), posted by SE-user V. Moretti
+ 4 like - 0 dislike

Your second equation is a misinterpretation of the notation in your first equation. $|q_1,q_2\rangle$ is not a vector in a finite dimensional vector space, having components $q_1$ and $q_2$ in some basis. Rather $|q_1,q_2\rangle $ and $|p_1,p_2,p_3\rangle$ are both vectors in the same infinite-dimensional Hilbert space.

In the simplest case, where all particles are bosons of the same species, this Hilbert space is spanned by the following collection of 'in' vectors.

  1. $|0\rangle$ -- a single vector of unit norm, representing the state with no incoming particles.
  2. $|p\rangle$ -- a unit vector for each momentum $p$. If $p'$ and $p'$ are different momenta, then $|p\rangle \neq |p'\rangle$. These vectors are states where we have precisely one particle coming in from past infinity with a fixed momentum.
  3. $|p_1,p_2\rangle$ -- a unit vector for each pair $(p_1,p_2)$ of momenta. $|p_1,p_2\rangle$ is not $|p_1',p_2'\rangle$ unless $p_1 = p_1'$ and $p_2=p_2'$ or $p_1 = p_2'$ and $p_2=p_1'$. These are states where we have two particles coming in from past infinity.
  4. $|p_1,p_2,p_3\rangle$ -- a unit vector for each triplet $(p_1,p_2,p_3)$. These vectors are only equal to each other if their labels $(p_1,p_2,p_3)$ are permutations of each other.
  5. ...

This is a continuous basis, which can seem a little weird if you're used to finite dimensional vector spaces. Finite sums get replaced by sum over number of particles and integrals over the momentum labels.

There's a similar basis $|0\rangle, \{|q\rangle\}, \{|q_1,q_2\rangle\},..$ of 'out' vectors, where now our momentum labels states according to the number of particles at future infinity with fixed momenta.

The S-matrix is the change of basis coefficients for switching from the in-basis to the out-basis.

This post imported from StackExchange Physics at 2014-04-24 02:32 (UCT), posted by SE-user user1504
answered Jan 18, 2014 by user1504 (1,110 points) [ no revision ]
Thanks, this is kind of the answer that is most on my level of current understanding and is exactly what I needed!

This post imported from StackExchange Physics at 2014-04-24 02:32 (UCT), posted by SE-user Hunter
I should also comment that many authors use a convention in which the vectors aren't normalized to have length 1. This is done purely for convenience; it has no physical content, since you can represent a state using any non-zero vector in the ray.

This post imported from StackExchange Physics at 2014-04-24 02:32 (UCT), posted by SE-user user1504
It may aid your learning to think about how @V.Moretti's answer generalizes mine. (For example: Imagine a system where particles can be of either of 2 species. Now you need to keep track of whether the momentum is for a particle of type 1 or type 2.)

This post imported from StackExchange Physics at 2014-04-24 02:32 (UCT), posted by SE-user user1504
+ 1 like - 0 dislike

What you seem to be thinking of is the "leading" comtribution to the amplitude -- where the each in-particle goes to an out-particle without interacting. This happens in any free/linear theory of waves passing through each other without interacting.

What we're interested in is the $T$ matrix in $S = \mathbf{1} + i T$ while you're talking about contribution from the $\mathbf{1}$. Imagine a simple "interaction" where one particle decays to a pair of particles. Such an aspect of the theory (among other possibilities) will contribute to the $T$ matrix and give you non-zero amplitudes even when the number of particles changes (maybe increases by one).

To put it another way, there is no reason that the number of particles ought to be conserved in a process, except in maybe special cases. Can you think of any symmetry that might imply such a conserved quantity?


Mathematically, in the perspective of states in a Hilbert space:

Without getting into deep questions about the "existence" of a Hilbert space for an interacting theory (which I don't know enough to comment on)... For an interacting theory, the space of states is the Fock space $\mathcal{H} = \oplus H_n$ where $H_n$ is the n-particle Hilbert space. Each of those states can be denoted by their quantum numbers under symmetries of the theory (representation theory).

$\langle q_1, q_2 | p_1, p_2 ,p_3 \rangle$ is NOT like $(q_1, q_2) \cdot (p_1, p_2, p_3)$. You should think of it as an inner product of some two states $\langle \Psi_2 | \Psi_3 \rangle$. $|\Psi_2 \rangle$ happens to be a certain state in the Fock space, parametrized by the two quantum numbers $q_1 , q_2$ (similar interpretation for $|\Psi_3 \rangle$). Each of those are states in an infinite dimensional vector/Hilbert/Fock space whose degrees of freedom can be thought of as excitations at each spatial point, for each field in the theory.

This post imported from StackExchange Physics at 2014-04-24 02:32 (UCT), posted by SE-user Siva
answered Jan 18, 2014 by Siva (720 points) [ no revision ]
From a physical point of view, I completely understand that the particles can change when they interact with each other (as has been determined by many experiments). My questions is more related to the mathematics of the Hilbert space and the inner product.

This post imported from StackExchange Physics at 2014-04-24 02:32 (UCT), posted by SE-user Hunter
Without getting into deep questions about the "existence" of a Hilbert space for an interacting theory (which I don't know enough to comment on)... For an interacting theory, the space of states is the Fock space $\mathcal{H} = \oplus_n H_n$ where $H_n$ is the n-particle Hilbert space. Does that help?

This post imported from StackExchange Physics at 2014-04-24 02:32 (UCT), posted by SE-user Siva
Hmmm, I am not sure. The Fock space I believe is spanned by: $$\{ |\mathbf{p}_1\rangle, |\mathbf{p}_1,\mathbf{p}_2\rangle,|\mathbf{p}_1,\mathbf{p}_2,\mathbf{p}_3\rangle‌​, \ldots \}$$Therefore, it makes sense to think that it is somehow useful for the interacting space. I am now wondering if I am thinking my question is unnecessarily complicated and I should just accept the way the S-matrix is defined (especially because none of the sources, such as Pesking & Schroeder, even mention this problem I am having).

This post imported from StackExchange Physics at 2014-04-24 02:32 (UCT), posted by SE-user Hunter
$\langle q_1, q_2 | p_1, p_2, p_3 \rangle$ is NOT like $(q_1 q_2) \cdot (p_1, p_2, p_3)$. You should think of it as an inner product of some two states $\langle \Psi_2 | \Psi_3 \rangle$. $| \Psi_2 \rangle$ happens to be a certain state in the Fock space, parametrized by the two quantum numbers $q_1, q_2$ (similar interpretation for $|\Psi_3 \rangle$). Each of those are states in an infinite dimensional vector/Hilbert/Fock space whose degrees of freedom can be thought of as excitations at each spatial point, for each field in the theory.

This post imported from StackExchange Physics at 2014-04-24 02:32 (UCT), posted by SE-user Siva
Ahhh ok, I now see what you mean. That does help me. Thanks! I will wait to accept your answer for a bit just to see if someone else will write something.

This post imported from StackExchange Physics at 2014-04-24 02:32 (UCT), posted by SE-user Hunter
Sure; I'll gather stuff from my comments and move it to the "answer".

This post imported from StackExchange Physics at 2014-04-24 02:32 (UCT), posted by SE-user Siva

Your answer

Please use answers only to (at least partly) answer questions. To comment, discuss, or ask for clarification, leave a comment instead.
To mask links under text, please type your text, highlight it, and click the "link" button. You can then enter your link URL.
Please consult the FAQ for as to how to format your post.
This is the answer box; if you want to write a comment instead, please use the 'add comment' button.
Live preview (may slow down editor)   Preview
Your name to display (optional):
Privacy: Your email address will only be used for sending these notifications.
Anti-spam verification:
If you are a human please identify the position of the character covered by the symbol $\varnothing$ in the following word:
$\varnothing\hbar$ysicsOverflow
Then drag the red bullet below over the corresponding character of our banner. When you drop it there, the bullet changes to green (on slow internet connections after a few seconds).
Please complete the anti-spam verification




user contributions licensed under cc by-sa 3.0 with attribution required

Your rights
...